Download as pdf or txt
Download as pdf or txt
You are on page 1of 55

Industrial Organization Markets and

Strategies 2nd Edition Belleflamme


Solutions Manual
Visit to download the full and correct content document: https://testbankdeal.com/dow
nload/industrial-organization-markets-and-strategies-2nd-edition-belleflamme-solution
s-manual/
Industrial Organization: Markets and Strategies
Paul Belle‡amme and Martin Peitz
published by Cambridge University Press

Part VI. Theory of competition policy


Exercises & Solutions

Exercise 1 Industries with cartels

Brie‡y describe and analyze a case of your choice concerning a price- or


quantity-…xing cartel (please not OPEC). The following questions may be useful
to bear in mind: What are the relevant characteristics of the industry? What
was the scope of the cartel? How was the cartel enforced? What were the e¤ects
of the cartels? How did the competition authority or court argue and what was
the decision, if any?

Exercise 2 Collusion and pricing

Two (advertising-free) newspapers compete in prices for an in…nite number


of days. The monopoly pro…ts (per day) in the newspaper market are M and
the discount rate (per day) is . If the newspapers compete in prices, they both
earn zero pro…ts in the static Nash equilibrium. Finally, if the …rms set the same
price, they split the market equally and earn the same pro…ts.

1. The newspapers would like to collude on the monopoly price. Write down
the strategies that the newspapers could follow to achieve this outcome.
Find the discount rates for which they are able to sustain the monopoly
price using these strategies.
2. On Sundays, the newspapers sell a weekly magazine (that can be bought
without buying the newspaper). The monopoly (competitive) pro…ts when
selling the magazine are also M (zero).
3. For which discount rates can the monopoly price be sustained only in the
market for magazines? (Write down the equation that characterizes the
solution.) Compare the solution found in question 1 and 2 and comment
brie‡y.
4. For which discount rates can the monopoly price be sustained both in the
market for newspapers and in the market for magazines? (Write down the
equation that characterizes the solution.)

Exercise 3 Collusion and pricing II [included in 2nd edition of the book]

1
Consider a homogeneous-product duopoly. The two …rms in the market are
assumed to have constant marginal costs of production equal to c. The two
…rms compete possibly over an in…nite time horizon. In each period they simul-
taneously set price pi , i = 1; 2. After each period the market is closed down with
probability 1 .
Market demand Q(p) is decreasing, where p = minfp1 ; p2 g. Suppose, fur-
thermore, that the monopoly problem is well de…ned, i.e. there is a solution
pM = arg maxp pQ(p). If …rms set the same price, they share total demand with
weight for …rm 1 and 1 for …rm 2. Suppose that 2 [1=2; 1). Suppose that
…rms use trigger strategies and Nash punishment.

1. Suppose that = 0. Derive the equilibrium of the game.


2. Suppose that > 0 and = 1=2. Derive the condition according to which
…rm 1 and …rm 2 do not …nd it pro…table to deviate from the collusive
price pM .
3. Suppose that > 0 and > 1=2. Derive the condition according according
to which pM is played along the equilibrium path. Show that the condition
is the more stringent the higher .
4. Show that previous results in (3) also hold for any collusive price pC 2
(c; pM ).
5. Suppose that > 0 and > 1=2 and that …rm 1 can only adjust its price
every periods. Derive the condition according to which pM is played
along the equilibrium path. How does the time span in‡uence the con-
dition?

Solution to Exercise3
1. Standard Bertrand model, p1 = p2 = c. Equilibrium pro…ts are equal to zero.
2. Pro…t with collusive price pM is (1=2) M
=(1 ), where M
pM Q(pM ).
M
The deviation pro…t is . Thus, …rms do not have an incentive to deviate if
M M
(1=2) =(1 ) , which is equivalent to 1=2.
3. Consider …rm 2. The deviation pro…t is M . Pro…t with collusive price pM
is (1 ) M =(1 ). Thus, …rm 2 does not have an incentive to deviate if
M M
(1 ) =(1 ) , which is equivalent to . This shows that the
condition is more stringent the higher 1=2.
M
4. As follows from the inequality (1 )e=(1 ) e for any e 2 (0; ) the
same inequality holds.
1 M
5. Consider …rm 2. The deviation pro…t is M + M + ::: + . Pro…t with
M M
collusive price p is (1 ) =(1 ). Thus, …rm 2 does not have an incentive
to deviate if (1 ) M =(1 ) M
(1 )=(1 ), which is equivalent to
. This shows that the condition is more stringent the higher 1=2.

2
Exercise 4 Parallel pricing and evidence of collusion
A competition policy authority has noticed that the …rms in the Lysine
industry consistently charge very similar prices, and the suspicion is that they
are colluding. Do you think that parallel pricing is proof of collusion? If not,
what kind of evidence would you look for?

Solution to Exercise 4 The problem of using parallel pricing as evidence of


collusion is that …rms can set similar prices both when they are colluding and when
they are competing …ercely. For example: If …rms compete in prices and have symmetric
costs, the equilibrium is that all …rms charge the same price (= the marginal cost).
Another piece of evidence that would point in the direction of collusion is a very
high mark-up in the industry. This requires, however, knowledge of the …rms’ cost
structure. Often, direct evidence of collusion (e-mails, letters, etc.) is therefore needed
to prove that collusion has taken place.

Exercise 5 Collusion and quantity competition [included in 2nd edition of the


book]
Consider the following market: Two …rms compete in quantities, i.e., they
are Cournot competitors. The …rms produce at constant marginal costs equal
to 20. The inverse demand curve in the market is given by P (q) = 260 q.
1. Find the equilibrium quantities under Cournot competition as well as the
quantity that a monopolist would produce. Calculate the equilibrium pro-
…ts in Cournot duopoly and the monopoly pro…ts. Suppose that the …rms
compete in this market for an in…nite number of periods. The discount
factor (per period) is , 2 (0; 1).
2. The …rms would like to collude in order to restrict the total quantity
produced to the monopoly quantity. Write down grim trigger strategies
that the …rms could use to achieve this outcome.
3. For which values of is collusion sustainable using the strategies of ques-
tion (b)? [Hint: Think carefully about what the optimal deviation is.]

Solution to Exercise 5
1. Here, q1 = q2 = 80 and q m = 120. Furthermore, 1 = 2 = 6400 and
m
= 14400:
2. Start by setting qi = q m =2 = 60 in the …rst period. In the later periods, set
qi = 60 if q1 = q2 = 60 in all previous periods. Otherwise, set qi = qi = 80:
3. We have that Ri (qj ) = 120 qj =2. This
implies that the optimal deviation to
Deviation
qj = 60 is qi = 90. Hence, = (260 90 60 20)90 = 8100: We
conclude that collusion can be sustained if and only if
14400 6400 9
8100 + , .
2(1 ) (1 ) 17

3
Exercise 6 The European air cargo cartel [included in 2nd edition of the book]

Read the following press release of the European Commission (Brussels,


March 28, 2012): http://europa.eu/rapid/press-release_IP-12-314_en.htm. For
the sake of the exercise, we model the European air cargo market during the 72
months of the cartel existence as follows. First, only the 14 …rms associated in
the cartel were active on the market; second, these …rms were symmetric; they all
had the same constant marginal cost, c, for supplying airfreight services; third,
these …rms competed à la Cournot (i.e., by choosing the quantity of airfreight
services); …nally, the inverse demand for airfreight services (per month) was
given by p = a 2q, where q denotes the total quantity of airfreight services
supplied by the 14 …rms.
As written in the text, the Commission …ned the cartel members a total
of e169 million. Given that the cartel lasted for 72 months, this is roughly
equivalent to a …ne of e2.35 million per month. This …ne is meant to compensate
the European consumers for the surplus reduction that they su¤ered because of
the existence of the cartel.

1. Show that (a c) had to be equal to e3.89 million (per month) to justify


the …ne of e2.35 million per month imposed by the Commission.
2. Set (a c) to e3.89 million. Assuming that the cartel pro…ts were equally
shared among the 14 cartel members, show that any of these …rms would
have been better o¤ by unilaterally leaving the cartel (which would have
then counted only 13 members) and by acting independently. What does
this tell you about the stability of cartels? Discuss.
3. Continue to set (a c) to e3.89 million. Take now a tacit collusion per-
spective. Suppose that the 14 …rms were following a grim trigger strategy
and were expecting to continue to compete inde…nitely on that market.
Compute the minimum discount factor that allowed the 14 …rms to sustain
full collusion (i.e., to behave as a cartel).

Solution to Exercise 6
1. We recall from the analysis of the symmetric Cournot oligopoly of Chapter
3 (with demand given by P (q) = a bq and linear marginal cost c) that the
total quantity at the Nash equilibrium is q (n) = n (a c) = (b (n + 1)). As
for consumer surplus, it is equal to CS (n) = (b=2) (q (n)). The reduction
in consumer surplus is given by CS (14) CS (1), where CS (14) is the
surplus that consumer ould obtain if the 14 …rms were competing à la
Cournot and CS (1) is the surplus that they actually obtain when the
…rms collude (and act thus as a monopoly). Setting b = 2, we …nd
49 1 2
CS (14) CS (1) = 225 16 (a c) = 2:35 , a c = 3:89:

2. We also recall from Chapter 3 that the equilibrium pro…t is equal to


2 2
(n) = (a c) =(b (n + 1) ). Each cartel member receives a share 1=14

4
2
of the monopoly pro…t, i.e., (1) =14 = (a c) =(56b). If a …rm leaves
the cartel, then competition takes place between two decision-makers (the
cartel made of the remaining 13 …rms and the deviating …rm); so the
2
deviating …rm earns (2) = (a c) =(9b). As 9b < 56b, deviation is pro-
…table, meaning that the cartel is unstable. Note that we do not need a
speci…c value for b and (a c) to make that argument.
3. Again, the answer does not depend on the speci…c value of b and (a c)
(you should nevertheless check this!). We can refer to Section 14.2.1:
the minimal discount factor in the symmetric linear Cournot oligopoly is
Cour 2 2 Cour
min (n) = (n + 1) = n + 6n + 1 . Setting n = 14, one …nds min (14) =
225=281 ' 0:8.

Exercise 7 Cournot mergers: pro…tability and welfare properties

Consider a homogeneous good duopoly with linear demand P (q) = 12 q,


where q is the total industry output, and constant marginal costs c = 3.

1. Suppose that …rms simultaneously set quantities. Determine the equilibri-


um (price, quantities, pro…t, welfare).
2. The …rms consider to merge although their production costs are not a¤ec-
ted. Determine the solution to this problem. Is such a merger pro…table?
What are the welfare e¤ects of such a merger?
3. Suppose that the merger is e¢ ciency enhancing, leading to marginal costs
cm < c. What are the welfare e¤ects of such a merger. Do you possibly
have to qualify your answer in (2)?
4. Consider the possibility of …rm entry after the merger (the entrant produ-
ces at marginal costs c = 3 and has entry cost e). Suppose …rst that the
merger is not e¢ ciency-enhancing. Analyze such a market and comment
on your result (depending on the entry cost e). Suppose next that the
merger is e¢ ciency-enhancing, i.e. cm < 3. Depending on cm and e, when
is a merger pro…table? [Hint: Calculate pro…ts for cm = 1=2.]
5. Many countries scrutinize merger and sometimes block them (or impose
remedies)? Discuss which factors should make the courts or the competi-
tion authority more inclined to block a merger.

Exercise 8 Cournot mergers and synergies. [included in 2nd edition of the


book]

Consider a homogeneous-product Cournot oligopoly with 4 …rms. Suppose


that the inverse demand function is P (q) = 64 q.

5
1. Suppose that …rms incur a constant marginal cost c = 4. Characterize the
Nash equilibrium of the game in which all …rms simultaneously choose
quantity.
2. Suppose that …rms 1 and 2 consider to merge and that there are synergies
leading to marginal costs cm < c. Characterize the Nash equilibrium. At
which level cm (you may want to give an approximate number) are the
two …rms indi¤erent whether to merge?
3. Is such a merger that just makes the two …rms indi¤erent between merging
and non-merging consumer-welfare increasing?

4. At which level cm would the merger be consumer-welfare neutral?


5. Suppose that instead …rms 1, 2, and 3 consider to merge. The new marginal
cost of the merged …rms is cn < c. At which level cn are the three …rms
indi¤erent whether to merge?
6. Compare your …ndings in (5) and (2). What can you say about incentives
to merge in this case?

Solution to Exercise 8
1. Since all …rms have the same
h costs, they all choosei the same quantity in equi-
P
librium: q = arg maxqi (60 qi j6=i qj )qi ) q = 12, p = 16 and
= 144.
2. The merged …rm sets qm = arg maxqm [(64 qm q3 q4 cm )qm ] while
the non-merged …rms set q3 = arg maxq3 [(60 qm q3 q4 )q3 ] and
q4 = arg maxq4 [(60 qm q3 q4 )q4 ]. By symmetry, q3 = q4 and optimal
quantities result as functions of cm : qm = 18 43 cm and q3 = q4 = 14 + 14 cm .
2 2
It follows that m = 18 43 cm and 3 = 4 = 14 + 41 cm . Firms 1 and
3 2
2 are indi¤erent to merge if m =2 , 18 4 cm = 288 , cm 1:373.
3. Consumer welfare before the merger was CSc = 1152. After the merger, con-
2
(46 14 cm )
sumer surplus is equal to CSm = 2 . Plugging in cm = 1:373 yields
CSm = 1042:27 which is smaller than CSc . Hence, a merger that makes the
merging …rms just indi¤erent is detrimental to consumers.

4. The merger will never be consumer welfare neutral because CSc = CSm ,
cm = 8.
5. Proceed as in (2) but with only two players left (the merged entity and one
remaining competitor). The resulting optimal quanities are qn = 68 32cn for
the merged …rm and q4 = 56+c 3
n
for the non-merged …rm. The …rms will be
indi¤erent to merge when m = 3 , cn 2:82.

6
6. A merger of 3 …rms is already pro…table for a smaller amount of synergies than
a merger of 2 …rms. This is due to the fact that merger-speci…c synergies balance
two opposing e¤ects. First, the merger leads to an internalization of competition
between the merging …rms. Second, it has an e¤ect on the remaining outsiders’
quantity choice that leads to lower pro…ts of the merged entity. When 3 instead of
only 2 …rms merge, the outsiders’e¤ect is smaller and therefore smaller synergies
are needed to balance both e¤ects and make the merging …rms indi¤erent.

Exercise 9 Cournot mergers and demand

Consider the following Cournot merger game. The inverse demand function
is of the form
P (q) = a q
where > 0 and there are n …rms with constant marginal costs of production
c.

1. Discuss the shape of the inverse demand function depending on .


2. Determine the Cournot equilibrium pro…ts in this set-up.
3. Determine all n for which a single merger, i.e. going from n to n 1 …rms,
is pro…table.

Exercise 10 Cournot mergers and asset complementarity [included in 2nd edi-


tion of the book]

Consider an n …rm symmetric homogeneous product Cournot oligopoly.


Each …rm i has a physical asset Ki = K normalized to 1. Its cost function
1
is Ci (qi ; Ki = 1) = qi2 + 32 . The demand side
P of the market is given by an
inverse demand curve P (q) = 1 q where q = i qi .

1. Determine equilibrium price, quantity, and pro…t of the n …rm Cournot


model speci…ed above. What is the upper bound on n for all …rms in the
industry to make non-negative pro…ts?
2. Consider now a merger between the two …rms n and n 1. Suppose …rst
that physical assets cannot be sold and that the assets of one of the mer-
ging partners are liquidated at zero cost, i.e. the cost function of the mer-
1
ged …rm is Ci (qi ; 1) = qi2 + 32 . Determine equilibrium price, quantity, and
pro…t of the …rm Cournot model after the merger. For which n are mergers
pro…table? For which n are mergers consumer surplus increasing?

7
3. Consider again a two-…rm merger, but suppose now that the merged …rm’s
assets can be combined giving rise to the cost function Ci (qi ; Ki ) = K1i qi2 ,
i.e. the merged …rm n 1 has cost function Cn 1 (qi ; 2) = 21 qi2 + 16 1
. De-
termine equilibrium price, quantity, and pro…t of the …rm Cournot model
after the merger. For which n are mergers pro…table? For which n are
mergers consumer surplus increasing? [Hint: you may want to solve parts
3 and 4 together]
4. Consider once again a two-…rm merger, but suppose now that the mer-
ged …rm’s assets are complementary. More speci…cally, suppose that the
1
merged …rm n 1 has cost function Cn 1 (qi ; 2) = 41 qi2 + 16 . Determine
equilibrium price, quantity, and pro…t of the …rm Cournot model after
the merger. For which n are mergers pro…table? For which n are mergers
consumer surplus increasing? What happens when the complementary is
even stronger?
5. Based on your …ndings in parts 2-4 what are the policy conclusions for an
antitrust authority?

Solution to Exercise 10
3 1 2 1
1. The answers are p (n) = n+3 , q (n) = n+3 , and (n) = (n+3) 2 32 . Hence,
2 2
(n) 0 if and only if (n+3)2 64 or equivalently n 5.

2. Firms are still symmetric after the merger. There are now n 1 …rms left in the
3 1
market and one compute p (n 1) = n+2 , q (n 1) = n+2 , and (n 1) =
2 1 2 1
(n+2)2 32 . A merger is pro…table if (n 1) > 2 (n), i.e. (n+2) 2 32 >
2 1
2( (n+3)2 32 ). It is easily checked that a merger in an industry with 3 or
more …rms is pro…table, whereas a merger from duopoly to monopoly is not
pro…table. Thus a merger to monopoly is non-pro…table whereas mergers in less
concentrated industries are. This non-standard result holds because of the high
cost incurred by a single …rm. Since p is decreasing in n, all mergers are consumer
surplus decreasing. Any pro…table merger should be prohibited.

3. The merged …rm n 1 is di¤erent from all other …rms i = 1; :::; n 2. Looking for
an equilibrium in which all non-merged …rms set the same quantity (equilibrium
values are denoted by superscript ), we can rewrite the system of …rst-order
conditions of pro…t maximization (with marginal costs of the merged …rm being
equal to qn 1 ) as a two equation system:

1 2qn 1 (n 2)qi = qn 1;
1 qn 1 (n 1)qi = 2qi :
3
For = 1, we obtain post-merger equilibrium values : qn 1 = 2n+5 , qi =
2 6 27 1
2n+5 (i = 1; :::n 2), p = 2n+5 and n 1 = 2(2n+5)2 16 . A merger
between …rm n and n 1 is pro…table if n 1 > 2 (n), which is equivalent
27 4
to 2(2n+5) 2 > (n+3)2 . Thus a 2-…rm merger in an n-…rm industry is pro…table

8
if 5n2 + 2n + 43 > 0 which is satis…ed for n up to n = 3. In other words,
mergers to monopoly and to duopoly are pro…table in this example. The merger
to duopoly increases the price from 1=2 to 6=11 and is therefore consumer
surplus decreasing. Any pro…table merger should be prohibited.

4. First-order conditions from part c with = 1=2 apply. We denote equilibrium


2
values after the merger by superscript . We compute: qn 1 = n+3 , qi =
1 3 6 1
n+3 (i = 1; :::n
2), p = n+3 and n 1 = (n+3) 2 16 . A merger
between …rm n and n 1 is pro…table if n 1 > 2 (n), which is always
satis…ed. Here, because of strong complementarities between assets, a merger
by two …rms is always pro…table. The level of complementarity here is a knife-
edge case in which consumer surplus is not a¤ected, as the price is the same as
prior to the merger. Hence, the competition authority can approve any proposed
merger on the grounds that the merger does not reduce consumer surplus. If
the complementarity is even stronger the merger is strictly consumer surplus
increasing.

5. If complementarities between merging …rms are su¢ ciently strong such that the
post-merger price is lower than the pre-merger price, the competition authority
should approve any proposed merger based on the consumer surplus criterion.
Depending on the cost function of the merged …rm a possible merger may not
be pro…table and, therefore, will never be proposed, be pro…table but consumer
surplus decreasing, or be pro…table and consumer surplus increasing. To dis-
tinguish between the last two cases, the competition authority must be able to
predict the post-merger price. To be able to do so, the merged …rm must present
evidence on the complementarity between the two assets.

Exercise 11 Mergers and free entry [included in 2nd edition of the book]
Consider a homogeneous product market with in…nitely many quantity-
setting …rms. The inverse demand function is given by P (q) = a q where
q is industry quantity. The cost function of each …rm is Cs (qi ) = cs qi + Fs for
qi > 0 and Cs (0) = 0. Suppose that parameters are such that, in any equi-
librium, more than one …rm is active (Note: In the analysis below it is NOT
required to derive the parameter restriction which guarantees that this property
is satis…ed.) For simplicity, the analysis below should be carried out under the
assumption that the number of …rms is a real number.
1. Characterize the set of pure-strategy free-entry equilibria of the game in
which all …rms simultaneously quantities. Determine equilibrium quantity
of each active …rm, equilibrium price, industry quantity, number of …rms,
and consumer welfare in equilibrium.
2. Consider a single merger between two …rms. Suppose that the merged …rm
has cost function Cm (qi ) = cm qi + Fm for qi > 0 and Cm (0) = 0 and that
cm = cs , while Fm 2Fs . Derive the exact condition when a merger is
pro…table when there is free entry before and after the merger.

9
3. In the setting of (2) is a pro…table merger welfare-increasing? Or is a
pro…table merger welfare-decreasing? Explain your …ndings.
4. Consider now a single merger after which the merged …rm has costs with
cm cs and Fm = Fs . Derive the exact condition when a merger is
pro…table when there is free entry before and after the merger.
5. In the setting of (4) is a pro…able merger welfare-increasing? Or is a pro-
…table merger welfare-decreasing? Explain your …ndings.

Solution to Exercise 11
1. Pro…t of …rm i is i = (a qi q i cs )qi Fs . FOCs can be written as qi =
(a q i cs )=2. In symmetric equilibrium with n …rms, we must have qi = qi
and q i = (n 1)qi . Using the FOC of …rm i, we obtain qi = (a cs )=(n + 1).
In free-entry equilibrium,
2
a cs
i = Fs = 0:
n+1
Solving for n we obtain that, in free-entry equilibrium,
a cs
n= p 1:
Fs
p p
We
p have that q = nqi = a cs Fs , p = cs + Fs , CS = (a cs
Fs )2 =2.
2. The best response function of the merged …rm is the same as non-merged …rms.
Hence, we obtain the same aggregate output and the same total number of active
…rms (one additional …rm enters as a result of the merger). The only di¤erence
is that the merged …rm has di¤erent …xed costs. Clearly, the merger is pro…table
if and only if Fm < Fs as in this case pro…ts increase from zero to Fs Fm :

3. A pro…table merger is always welfare-increasing. Consumer surplus is not a¤ec-


ted by the merger, the pro…ts of outsiders are zero before and after the merger.
Thus the only change in welfare is due to the change of the …rms involved in the
merger.

4. Denote the newly merged …rm as …rm 1. Then q1 = (a q 1 cm )=2 and


qi = (a q i cs )=2 P for i 6= 1. For the non-merged …rms, in free-entry
equilibrium, (P (q1 + i6=1 qi ))qi = Fs . Since the best response of …rm i and
the inverse demand function P are the same before and after the merger, in
free-entry equilibrium after the merger the output of active outsider …rms is
the same as before the merger. In addition total industry output q remains
the same, which implies that consumer surplus is unchanged. Using the FOC of
…rm 1, we know that q1m = (a (q q1m ) cm )=2 after the merger. Hence,
m
q1 = a q cm which is larger than the quantity of a single …rm prior to the
merger, q1 , since marginal costs have been decreased due to the merger. Since

10
the merger does not a¤ect aggregate output, we have q1m q1 = cs cm . The
merger is pro…t-increasing if and only if cs > cm .

5. Since consumer surplus and outsider pro…ts are not a¤ected by the merger, the
merger is welfare-increasing if and only if it is pro…table. The …ndings illustrate
that barriers to entry are at the heart of merger analysis, as private and social
incentives are aligned under free entry.

Exercise 12 Burning ships


Hernan Cortéz, the Spanish conqueror (”conquistador”), is said to have bur-
ned his ships upon the arrival to Mexico. Why would he do such a thing?

Exercise 13 Quantity commitment [included in 2nd edition of the book]


Up to two …rms are in a market in which quantities are the strategic variable.
There are two periods; in the …rst period …rm 1 is a protected monopolist.
In each of the two periods t = 1; 2 the inverse demand function P t is given
by P t (xt ) = 20 xt . In each period the cost function of …rm i is given by
Cit (xti ) = 9 + 4xti . Pro…ts of a …rm are the sum of its pro…ts in each period (no
discounting). Firms maximize pro…ts by setting quantities.
1. Determine the monopoly solution.
2. Because of technological restrictions …rm 1 has to choose the same quantity
in each period (x11 = x21 ). Observing x11 , …rm 2 is considering to enter in
period 2. Determine the pro…t maximizing x22 given x11 .
3. Assume that …rm 2 will enter in period 2. What quantity will …rm 1
produce? Determine equilibrium prices, quantities, and pro…ts.
4. Firm 2 only enters in period 2 if it can make positive pro…ts. Determine
the subgame perfect equilibrium of the two-period model.

Solution to Exercise 13
1. In the monopoly situation, we can analyze a single period (both periods are
identical). The …rm chooses q1 to maximize t1 = (20 q1 )q1 9 4q1 . The
…rst-order condition is 16 2q1 = 0. Thus, q1 = 8, P = 12 and t1 = 55. In
consequence, total pro…ts are 1 = 2 t1 = 110.
t=2
2. Firm 2’s pro…t given q1 is 2 = (20 q1 q2 )q2 9 4q2 . We …nd …rm 2’s
1
best response from the …rst-order condition: q2 = 8 2 q1 . When playing this
best reposnse, …rm 2 achieves the following pro…t:
t=2 1 1 1
2 = (20 q1 (8 2 q1 ))(8 2 q1 ) 9 4(8 2 q1 )
1
= 4 (q1 10) (q1 22) :
As q2 < 0 for q1 > 22, we see that the condition for …rm 2 to make a positive
pro…t (and thus to enter) is q1 10.

11
3. Assuming that …rm 2 enters, …rm 1 has to consider both pro…ts on the …rst and
second period in its decision (as q1 can not be changed between the periods per
assumption):
t=1 t=2
1 = 1 + 1 = (20 q1 )q1 9 4q1 + (20 q1 q2 )q1 9 4q1 :
1
Now, we can set in q2 = 8 2 q1 as …rm 1 knows how …rm 2 will react on a
given q1 ; doing so, we have 1 = 32 16q1 q12 12 . The …rst-order condition
yields 16 2q1 = 0 , q1 = 8. As a result, q2 = 4, P 1 = 12, P 2 = 8, 1 = 78
and 2 = 7.

4. To …nd the subgame-perfect equilibrium, we need to be determined whether


the incumbent …rm 1 would prefer to accommodate entry of …rm 2 in period
2 or deter entry. Given accommodated entry, we know from (3) that 1 = 78.
To deter entry, we know from (2) that …rm 1 then needs to set q1 = 10 (or
higher). We thus determine the pro…t for …rm 1 under this strategy: 1 (q1 =
10) = 2 [(20 q1 ) q1 9 4q1 ] = 102.Thus, pro…ts for …rm 1 under entry
deterrence are higher than under accommodated entry.

Exercise 14 Strategic quantity choice

Consider a market with two …rms, A and B. The …rms produce homogenous
goods, compete in quantities, and face a constant marginal cost equal to 1/4.
The timing is the following: First, …rm A chooses its quantity qA . Then, after
observing qA , …rm B chooses its quantity qB . The price in the market is given
by the inverse demand function P (q) = 1 q, where q = qA + qB .

1. Find the subgame perfect Nash equilibrium.


2. Assume from now that there is an entry cost of e. Firm A is already
established in the market, and …rm B is considering whether to establish
itself in the market or not. The timing is the following: First, …rm A
chooses its quantity qA . Then, after observing qA , …rm B decides whether
to enter the market and, in case of entry, how much to produce.
3. Write down …rm B’s pro…t function.
4. Assume for now that e = 1=10. Illustrate …rm A’s pro…t as a function of
qA when the reaction of …rm B is taken into account.
5. Find the subgame perfect Nash equilibrium for all values of e > 0.

Exercise 15 Taxonomy of entry-related strategies I

12
Consider a market with di¤erentiated products. In the …rst stage …rm 1 is
the incumbent …rm and can invest an amount K1 0 in reducing its marginal
costs, c(K1 ) = c K1 =10. In stage two …rm 2 can decides about entering the
market with constant marginal costs of c and entry costs of e. In stage three if
entry takes place …rms engage in price competition and face symmetric demand
functions given by Di (pi ; pj ) = A api + bpj (A > a > b > 0). If no entry takes
place, …rm 1 acts as a monopolist with demand, D1 (p1 ) = A ap1 .
1. Calculate the best response functions for both …rms. Draw a graph.
Argue graphically from now on:
2. Does an increase in K1 increase or decrease the pro…t of the entering …rm?
Does an increase in K1 make the incumbent tough or soft?
3. Does a marginal investment K1 increase or decrease the pro…t of the in-
cumbent? (Assume that e is su¢ ciently low such that entry takes place
for K1 close to zero. Moreover, assume A 10)
4. Use your answer of (2): Is entry deterrence via cost reduction possible in
this setting? If your answer is YES, which numbers would you have to
compare to decide whether entry deterrence is optimal? If your answer is
NO, what do we have change in this model to induce entry deterrence?
5. Use your answer of (3): If entry accommodation is optimal how much
should …rm 1 invest in cost reduction?
6. How would you answer to (4) change if we consider a Cournot game ins-
tead?
7. How would you answer to (5) change if we consider a Cournot game ins-
tead?

Solution to Exercise 15
demand Di (Pi ; Pj ) = A api + bpj ; A>a>b>0
K1
incumbent can invest in cost reduction K1 0, M C = c(K1 ) = c 10

entrant faces M C = c and entry costs = e

1. Best responses:
max i (pi ; pj ) = (pi ci )(A api + bpj )
!
FOC: a(pi ci ) + A api + bpj = 0
2api = A + aci + bpj

A+aci +bpj
pi (pj ) = 2a …rm i’s best response

K1
c(K1 ) = c 10 ; if i = 1
and ci =
c; if i = 2

13
P2
P1* (P2)

P2* (P1)

P1

Abbildung 1: Strategic complements (prices)

Best-response functions and iso-pro…t lines (upper contour sets opened towards
the north east and tangent on vertical resp. horizontal line by FOC).

2. Graphical argument:
K1 "
A + a(c 10 ) b
p1 (p2 ) = #+ pj
2a 2a

K1 " ! p1 -intercept of …rm i’s best response function moves to the left

K1 "! 2 #

Alternatively:
z }| {
z}|{ z}|{ z }| {
d 2 @ 2 @ @p1 @c
jK1 =0 = + 2 <0
dK1 @K @p @c @K1
| {z 1} | 1 {z }
=0 stra te g ic e ¤ e c t <0

Hence, ivestment in cost reduction makes …rm 1 tough.

14
3.

z }| {
z }| { z}|{ z}|{ z}|{ z}|{
d 1 @ 1 @ 1 @p1 @c @ 1 @p2 @p1 c
jK =0 = + + <0
dK1 1 @K @p1 @c @K1 @p2 @p1 @c K1
| {z 1} |{z} | {z }
d ire c t e ¤ e c t ! 0 by E nve lo p e T h . = 0
; strategic e¤ect

4. Entry deterrence (D):

d 2
Is possible here: Since < 0 (see b)); 9 K1D > 0; s.t. D
2 (K1 ) =e
dK1
M onopolist
Entry deterrence is optimal, if 1 (K1D ) K1D D
1 (K1 )

5. Entry accommodation (A); see (3):


Since the strategic e¤ect is negative, …rm 1’s pro…t decreases in the level of cost
d
reduction, dK11 < 0. Thus, K1A = 0 is optimal if entry occurs.

6. Entry deterrence under Cournot: Entry deterrence is possible because the de-
terrence level of cost reduction is also positive, i.e. K1D > 0. Entry deterrence
is optimal under Cournot if M D
1 (K1 ) K1D 1
Cournot

7. Entry accommodation under Cournot: K1A > 0 is optimal here, because the
strategic e¤ect is positive under Cournot (and we implicitly assume it to be
larger than the direct e¤ect)

z }| {
z }| { z}|{ z}|{ z}|{ z }| {
d 1 @ 1 @ @q2 @q1 @c
jK1 =0 = + 1 >0
dK1 @K @q @q1 @c @K1
| {z 1} | 2 {z }
d ire c t e ¤ e c t ! 0
; strategic e¤ect

Best-response functions and iso-pro…t lines (upper contour sets opened towards
the south resp. west and tangent on vertical resp. horizontal line by FOC)

Exercise 16 Taxonomy of entry-related strategies II

15
q2 c1

q1* (q2)

q1* (q2)

q2*(q1)
_
π1
q1
qM

Abbildung 2: Strategic substitutes (quantities)

Consider the market from the previous exercise again.

1. Use the best-response functions from the previous exercise to calculate


equilibrium prices for c1 6= c2 in the Nash equilibrium in which …rms set
prises.
2. Use your result from (1) to show that pi = (A + ac)=(2a b), for i = 1; 2,
if c1 = c2 = c.

Now, use again that c1 (K1 ) = c (K1 =10) and c2 = c.


Set c = 4, a = 2, b = 1, A = 10, and e = 7:95.

3. Show that the critical level of K1D to deter entry is below 0:5.
4. Suppose that investment in cost reduction is restricted to half units, i.e.
K1 2 f0; 0:5; 1; 1:5; : : :g. Will …rm 1 deter entry in a subgame perfect Nash
equilibrium? State …rm 1’s optimal business strategy.
5. Reconsider your answer to (4) if …rm 1 as a monopolist faces a demand

of D1 (p1 ) = A ap1 + bp1 = A (a b)p1 .

16
Solution to Exercise 16
1. obvious
2. c1 = c2 ) p1 = p2 :
2a(A + ac) + b(A + ac) (2a + b)(A + ac) A + ac
pi = = = Q:E:D:
(2a b)(2a + b) (2a b)(2a + b) 2a b

3. Entry deterrence if 2 0:

2 = (p2 c)(A ap2 + bp1 ) e 0


! plug in numbers:
1 2 4
(2 75 K1 )(10 12 + 75 K1 + 6 75 K1 ) 7:95 0
1 2
(2 75 K1 )(4 75 )K1 0
2 2 8
752 K1 75 K1 0:05
8K1
+0:05
K12 75
2 + 2 =0
752 752
K12 300K
p 1 + 140:625 = 0
K1=2 = 150 1502 + 140:625

! K1 = 0:4695
[K2 = 299:5305]

4. Compare pro…ts under entry determination and entry accommodation:


Accommodation case: From the previous exercise we know K1A = 0

1 = (p c)(A ap + bp)

! numbers: p = A+ac 10+8


2a b = 4 1 = 6
A
1 = 2 (10 12 + 6) = 8

Deterrence case: Use K1D = 0:5 from now on


Demand is now given by D1 (p1 ) = A ap1 . Note that this assumption is
not very plausible because it implicitly implies that p2 = 0 (the market is too
small with only one product, because consumer behave as if there exists a close
substitute for free). Thus, the bene…ts of entry deterrence are underestimated.

1 = (p 3:95)(10 2p) 0:5


@ 1 !
@p = 4p + 17:9 = 0

p = 4:475
D
1 = (0:525)(10 8:95) 0:5 = 0:05125

A D
Since 1 > 1 in SPNE …rm 1 will accommodate entry.

17
5.
D
1 = (p 3:95)(10 2p + p) 0:5
D !
@
@p
1
= 2p + 13:95 = 0

p = 6:975
D A
1 = (3:025)(10 6:975) 0:5 = 8:650625 > 1

Hence, …rm 1 deters entry in SPNE.

Exercise 17 Sequential quantity choice and entry

Consider a market for a homogenous good with one incumbent …rm (…rm
1) and one potential entrant (…rm 2). The interaction between the two …rms
evolves in two stages. In stage 1, …rm 1 chooses its quantity q1 . In stage 2, after
observing q1 , …rm 2 decides whether or not to enter the market. If it enters,
it incurs an entry cost e and chooses its own quantity, q2 . If …rm 2 does not
enter then q2 = 0 and …rm 2 does not pay the entry cost e (…rm 1 then is a
monopoly). Assume that the inverse demand for the good is P = a (q1 + q2 ),
and that the cost of production of each …rm i is C(qi ) = qi =2.

1. Compute the range of e for which entry is blockaded. That is, compute
…rm 1’s output when it operates as a monopolist, then given this quantity,
compute the highest pro…t that …rm 2 can earn if it decides to enter, and
…nally, compute the range of e for which entry is blockaded.
2. Now, suppose that e is su¢ ciently low to ensure that entry is not blo-
ckaded. Compute the quantities and pro…ts of each …rm when entry is
accommodated. That is, compute the outputs that will be selected in a
Stackelberg equilibrium and the resulting pro…ts. (Instruction: …rst, com-
pute …rm 2’s best response function, br2 (q1 ). Second, substitute for br2 (q1 )
into …rm 1’s pro…t function and compute …rm 1’s pro…t-maximizing quan-
tity q1 . Third, …nd …rm 2’s best response against q1 , using …rm 2’s best
response function. Finally, given the pair of quantities you found, compute
the equilibrium pro…ts).
3. Compute the lowest q1 for which entry is deterred. Compute …rm 1’s pro…ts
at this output level.
4. Given your answer in (3), show …rm 2’s best response function graphically
in the quantities space (recall that …rm 2 may wish to stay out of the
market when q1 is relatively high). Show on the same graph the Stackelberg
equilibrium you found in Section (3) and the lowest q1 for which entry is
deterred.

18
5. Given your answers in (2) and (3), …nd the range of e for which entry
is accommodated, and the range of e for which it is deterred. Explain
in no more than 3 sentences the intuition for the result (i.e., why is it
natural to expect that entry is accommodated/deterred when e is relatively
low/high).

Solution to Exercise 17
1. If …rm 1 is a monopolist it produces q1 = a=3. Standard calculations reveal
that …rm 2’s best response function is br2 (q1 ) = (a q1 )=3. Substituting this
expression in …rm 2’s pro…t gives that …rm 2’s pro…t, when it plays a best
response against …rm 1, is (a q1 )2 =6 e. For q1 = a=3, …rm 2’s pro…t is
2a =27 F . This implies that entry is blockaded for all e > 2a2 =27.
2

2. If entry is accommodated, then …rm 1 chooses q1 subject to q2 = br2 (q1 ), which


result in q1 = (2=7)a. Firm 2 then chooses q2 = (5=21)a. For these quantities,
…rm 1’s pro…t is A 2
1 = 2a =21.

3. and 4. As we saw in (1), …rm 2’s pro…t, provided that it plays a best-response against
…rm 1, is (a q1 )2 =6 e. Entry is deterred if this pro…t is less than or equal
to 0. Solving, the equation (a q1 )2 =6 e = 0, impliesp that to deter entry
…rm 1 must produce the deterring quantity q1D = a 6e. Note that, since we
assume that entry is not blockaded, e 2a2 =27, we have
r
D
p 12 2 1
q1 = a 6e a a = a;
27 3
where a=3 is the monopoly output of …rm 1. Hence, to deter entry, …rm 1
must produce strictly above its monopoly output and more so as e continues to
fall from 2a2 =27, which is the critical level of e that separates deterred from
blockaded entry. (For e 2a2 =27, entry is blockaded. Hence, …rm 1 can behave
D
p and 2simply ignore …rm 2.) Firm 1’s pro…t when it produces q1
as a monopolist
is D 1 = 2a 6e a =2 9e.

5. To solve this problem we need to compare the pro…t under accommodation


A
with the pro…t under deterrence. Firm 1’s pro…t under accommodation is 1
= 2a2 =21, which is clearly independent of e. Firm 1’s pro…t under deterrence
D
1 is increasing in e because the higher e is, the lower is the entrant’s pro-
…t, and, hence, the lower …rm 1’s quantity that is necessary to deter entry.
Hence, it is more pro…table to accommodate when e is relatively small and
to deter if e is relatively large. Solving thepequation A 1 =
D
1 we obtain
2
that entry
p is accommodated if e < (31 4 21)a =378 and it is deterred if
(31 4 21)a2 =378 < e < 2a2 =27.

Exercise 18 Capacity choice and entry [included in 2nd edition of the book]

19
Consider an industry for a homogenous product with a single …rm (…rm 1)
that can produce at zero cost. The demand function in the industry is given by
Q = a p. Now suppose that a second …rm (…rm 2) considers entry into the
industry. Firm 2 can also produce at zero cost. If …rm 2 enters, …rms 1 and 2
compete by setting prices. Consumers buy from the …rm that sets the lowest
price. If both …rms charge the same prices, consumers buy from …rm 1.
1. Solve for the Nash equilibrium if …rm 2 chooses to enter the industry.
Would …rm 2 wish to enter if entry required some initial investment?
2. Now suppose that before it enters, …rm 2 can choose a capacity, x2 , and a
price p2 (the capacity x2 means that …rm 2 can produce no more than q2 =
x2 units). Given q2 and p2 , …rm 1 chooses its price and then consumers
decide who to buy from. Compute the Nash equilibrium in the product
market if …rm 1 chooses to …ght …rm 2. What is …rm 1’s pro…t in this
case? Show …rm 1’s pro…t in a graph that has quantity on the horizontal
axis and price on the vertical axis. Would …rm 2 choose to produce in that
case?
3. Now suppose that …rm 1 decides to accommodate the entry of …rm 2.
Compute the residual demand that …rm 1 faces after …rm 2 sells q2 units,
and then write the maximization problem of …rm 1 and solve it for p1 .
What is …rm 1’s pro…t if it decides to accommodate …rm 2’s entry? Draw
…rm 1’s pro…t in a graph that has quantity on the horizontal axis and price
on the vertical axis. Would …rm 2 wish to enter in this case?
4. Given your answers to (2) and (3), compute for each p2 the largest capacity
that …rm 2 can choose without inducing …rm 1 to …ght it. (Hint: to answer
the question you need to solve a quadratic equation. The solution is given
by the small root).
5. Show that the capacity you computed in (4) is decreasing with p2 . Explain
the intuition for your answer. Given your answer, explain how …rm 2 will
choose its price. Computing p2 is too complicated; you are just asked to
explain in words how …rm 2 chooses p2 .)

Solution to Exercise 18
1. In a Nash equilibrium, both …rms will charge prices equal to 0. This is the only
pair of prices for which no …rm can bene…t from deviation. If prices are negative,
…rms lose money and are better o¤ charging 0 (in which case they do not lose
money). If prices are positive it pays to cut the price by a cent below the price of
the rival and, thereby, capture the entire market. Hence, if entry requires even
a small initial investment, …rm 2 will choose to stay out.

2. If …rm 1 …ghts it charges p2 and captures the entire market because when both
…rms set equal prices, all consumers prefer to buy from …rm 1. Firm 1’s pro…t
then is F 1 = (a p2 )p2 . Firm 2 obtains 0 pro…ts and, hence, would prefer to
stay out.

20
3. If …rm 1 accommodates …rm 2, its residual demand is Q1 = a q2 p1 . The
problem of …rm 1 is to maximize p1 Q1 . The price that maximizes …rm 1’s pro…t
is p1 = (a q2 )=2, so …rm 1’s pro…t is A 1 = (a q2 )2 =4. In this case, …rm 2
earns p2 q2 > 0 and hence would choose to enter.

4. We need to compare F A
1 and 1 . This comparison reveals that for each p2 ,
the largest x2 that
p …rm 2 can choose without inducing …rm 1 to …ght it is
x2 (p2 ) = a 2 (a p2 )p2 .
5. x2 (p2 ) is decreasing in p2 if p2 > a=2 and increasing otherwise. To determine
whether p2 is above or below a=2, note that the entrant’s pro…t is p2 x2 (p2 ).
If p2 < a=2, then since x2 (p2 ) is increasing in p2 , the entrant would want to
raise p2 as much as possible since both his capacity and his pro…t per unit will
increase. This happens until a=2. Thus, in equilibrium, it must be the case that
p2 > a=2 so that x2 (p2 ) is decreasing in p2 .

Exercise 19 Investment and incumbency

Consider a di¤erentiated product market. At the …rst stage …rm 1 is the


incumbent …rm and can invest an amount I1 0 in reducing its marginal
costs, c(I1 ) = c I1 =10. At stage two …rm 2 decides whether to enter the
market with constant marginal costs of c and entry costs of e, which is sunk
at this stage. At stage three, if entry has taken place, …rms engage in quantity
competition and face inverse demand functions given by Pi (qi ; qj ) = a bqi dqj
(a > b > d > 0). If no entry takes place, …rm 1 acts as a monopolist with inverse
demand, P m (q1 ) = a bq1 .

1. Calculate the best response functions for both …rms. Draw a graph.
2. Does an increase in I1 increase or decrease the pro…t of the entering …rm?
Does an increase in I1 make the incumbent tough or soft?
3. Does a marginal investment I1 > 0 increase or decrease the pro…t of the
incumbent? (Assume that e is su¢ ciently low such that entry takes place
for I1 close to zero.)
4. If entry accommodation is optimal how much should …rm 1 invest in cost
reduction? (Use your answer of (3).)
5. Is entry deterrence via cost reduction possible and pro…table in this set-
ting? Discuss your results in the light of the taxonomy developed in the
book.

Exercise 20 Competition and entry [partly included in 2nd edition of the book]

21
Consider a homogeneous good duopoly with linear demand P (q) = 1 q,
where q is the total industry output. Suppose that …rms are quantity setters
and …rms incur constant marginal costs of production ci .

1. Suppose that …rms have constant marginal costs of production c. Deter-


mine the Nash equilibrium in quantities (report prices, quantities, pro…t,
welfare)
2. Reconsider your answer in (1) because of the following: A tabloid runs a
series on consumers paying “excessive” prices. The government considers
introducing a non-negative special sales tax t 0 per unit on this pro-
duct (and plans to use the revenues for some project from which nobody
bene…ts). Determine the welfare-maximizing tax rate (the government is
assumed to be able to commit to the tax; welfare is total surplus which in-
cludes tax revenues). Discuss your result. What would be your conclusion
if the government was considering subsidizing the …rm?
3. Return to the case without taxes. Consider now the duopoly with c1 = 0
and c2 = c 2 [0; 1]. Determine the equilibrium (price, quantities, pro…t,
welfare).
4. Consider now an extended model in which only …rm 1 is necessarily pre-
sent. At stage 1, …rm 1 can make an investment I after which …rm 2’s
marginal costs is c2 = 1=2 instead of c2 = 0. Afterwards, …rm 2 observes
the investment decision of …rm 1 and, at stage 2, decides whether to enter
at a negligible entry cost e > 0. At stage 3, active …rms set quantities
simultaneously. Determine the subgame perfect equilibrium of this game.
Discuss your result in the light of what you have learnt reading about
entry-related strategies (max 3 sentences).
5. Consider now a di¤erent entry model. Both …rms have zero marginal costs
of production but consumers have become accustomed to product 1 (even
if they did not consume it themselves). Therefore, consumers are willing
to pay 1=2 money units less for product 2 than for product 1. The inverse
demand function P (q) = 1 q gives demand for product 1. At stage 1,
the potential entrant, …rm 2, considers to enter the market at an entry
cost e > 0. (There is no investment stage in this game.) At stage 2,
…rms set quantities simultaneously. Report the pro…t function for each
…rm. Determine the equilibrium in case …rm 2 has entered (report prices,
quantities, pro…t, welfare). Determine the subgame perfect equilibrium
and comment on your result. You may want to reuse some of the results
derived above.

Solution to Exercise 20
1. maxqi (1 q c)qi
foc: 1 2qi qj c = 0.

22
symmetric equilibrium: 1 3qi c = 0.
qi = (1 c)=3, q = 2(1 c)=3
p = P (q ) = 31 + 23 c
2
= 13 c
R1 p ]2 2
CS = p (t)
qdp = [1 2 = 92 (1 c)
T S(t) = 2 + CS = 94 (1 c)2

2. maxqi (1 q t c)qi
q (t) = 2(1 c t)=3
p (t) = P (q ) = 31 + 32 (c + t)
p (t) c t = 1 3c t
2
(t) = 91 (1 c t)
Z 1 2
[1 p (t)]
CS(t) = qdp =
p (t) 2
2 2
3 (1 c t)
=
2
2
= (1 c t)2
9

T S(t) = 2 (t) + CS(t) + tq (t)


2 2 2
= (1 c t)2 + (1 c t)2 + t (1 c t)
9 9 3

dT S(t) 8 2 2
= (1 c t) + (1 c t) t
dt 9 3 3
< 0

The optimal tax is zero. The government should not impose a tax as a response
to market power.

3. Suppose that …rm 2 is active:


1 q2
foc …rm 1: 1 2q1 q2 = 0. Hence, q1 = 2 :
foc …rm 2: 1 c q1 2q2 = 0. Hence, q2 = 1 c2 q1 :
Substitute from above:
1 q2
1 c 2q2 = 0
2
1 2
q2 = c
3 3

23
1
Substitute back: q1 = 3 + 31 c.

1 1 1 2
p = P (q ) = 1 [ + c+ c]
3 3 3 3
1 1
= + c
3 3
1
1 = (1 + c)2
9
1
2 = (1 2c)2
9
Equilibrium demand q2 positive as long as c 1=2. For higher marginal costs
q2 = 0 and …rm 1 maxq1 (1 q1 c)q1 under the constraint that the best
response of …rm 2 is zero, i.e. 1 c2 q1 = 0. Solving for q1 in the constraint gives
q1 = 1 c. P (q1 ) = c in this case. Firm 1’s pro…t is c(1 c). The unconstrained
problem gives q1 = 1=2. Hence, the constraint is not binding for c > 1=2 and
the …rm sets the unconstrained monopoly price (i.e. …rm 1 does not want to set
a quantity lower than 1=2 in monopoly).

4. Solve by backward induction. The investment will lead to q2 = 0. Thus …rm 1


makes (monopoly) pro…t 1=4. If …rm 1 does not invest it will make pro…t 1=9
1 c 2
(take solution from (1): 3 ). Hence the investment is pro…table as long as

1 1
I
4 9
or I 5=36. Note that the investment here raises the rival’s costs. However,
the strategy is here NOT used as a deterrence strategy because …rm 2 would
not be active in any case (even if it does not have to decide whether to enter).

5. Given q1 and q2 , prices are p1 = P (q1 + q2 ) = 1 q1 q2 and p2 = P (q1 +


q2 ) 1=2 = 1=2 q1 q2 .
FOC …rm 1: 1 2q1 q2 = 0.
FOC …rm 2: 1=2 q1 2q2 = 0.
Use analysis from part (3) with c = 1=2. Hence, using the formulas from above,
…rm 2 does not have an incentive to enter the market since it would make zero
sales. The presence of switching costs leads to zero quantity by …rm 2. This
would even hold if entry was not costly. Here, switching costs do not lead to
entry for any level of entry costs. For non-negligible entry costs, entry can even
be avoided for lower levels of the switching cost.
The equilibrium values are q2 = 0, q1 = 1=2, p = 1=2, 1 = 1=4, 2 = 0,
CS = 1=8 and T S = 3=8.

Exercise 21 Upstream merger

24
Consider an industry with two symmetric upstream …rms A and B and one
downstream …rm D. The downstream …rm may sell the product of none, one,
or both upstream …rms. Total industry pro…ts are a function of the number
of upstream …rms selling through D, denoted by V (n), n 2 f0; 1; 2g, which is
assumed to be increasing in n. The outside option for each …rm of not selling is
zero. Thus, V (0) = 0.
Rents are the outcome of Nash bargaining between any pair of one upstream
…rm and the downstream …rm (equal sharing of the surplus within the pair above
the pro…ts that would occur if this pair did not agree).

1. What will be the pro…ts of …rms A, B and D?


2. Suppose that the two upstream …rms merge (and become a two-product
…rm). Thus Nash bargaining takes place between the merged upstream
…rm and the downstream …rm. What will be the pro…ts of the merged
upstream …rm AB and the downstream …rm D?
3. Provide the exact condition for the merger to be pro…table.
4. Explain your …ndings.

Solution to Exercise 21
1. Suppose that there is an agreement between B and D. If A and D do not reach
an agreement, neither A nor D obtain any pro…t from product 1. The additional
industry pro…t generated by the agreement between B and D is V (2) V (1).
This gain is shared equally and thus B = [V (2) V (1)]=2. Symmetrically,
A = [V (2) V (1)]=2. Hence, D = V (2) A B = V (1).

2. The industry pro…t is shared equally between both …rms, AB = V (2)=2 and
D = V (2)=2.

3. The merger is pro…table if AB > A + B, which is equivalent to V (2)=2 >


V (2) V (1) or V (1) > V (2)=2.
4. Prior to the merger, any pair with one upstream and the downstream …rm
bargain under the presumption that the other upstream …rm sells through the
downstream …rm. This leads to lower upstream …rm pro…ts than after the merger
if V is concave in n.

Exercise 22 Non-linear pricing in the supply chain

A monopolist produces a good with constant marginal cost equal to c, c <


1. Assume for now that all consumers have the demand Q(p) = 1 p. The
population is of size 1.

25
1. Suppose that the monopolist cannot discriminate in any way among the
consumers and has to charge a uniform price, pU . Calculate both the price
that maximizes pro…ts and the pro…ts that correspond to this price.
2. Suppose now that the monopolist can charge a two-part tari¤ (m; p) where
m is the …xed fee and p is the price per unit. Expenditure then is m + pq.
Calculate the two-part tari¤ that maximizes pro…ts and the pro…ts that
correspond to this tari¤. Compare pU and p and comment brie‡y.
3. Compare the situation with a uniform price and a two-part tari¤ in terms
of welfare (a verbal argument is su¢ cient).

4. Assume now instead that there are two types of consumers. The consumers
of type 1 have the demand Q1 (p) = 1 p, and the consumers of type 2
have the demand Q2 (p) = 1 p=2. The population is of size 1 and there
are equally many consumers of the two types. Finally, it is assumed in this
question that c = 1=2. Calculate the two-part tari¤ that maximizes the
pro…ts of the monopolist. Compare the two-part tari¤s found in questions
(2) and (3) for c = 1=2 and comment brie‡y.

Exercise 23 RPM

RPM was common in a number of industries. In particular, it could be


observed in the clothing, consumer electronics, and food industry. What is the
probable motivation for …rms to use RPM in these industries? Discuss the likely
welfare consequences in these industries.

Exercise 24 Vertical contracting [included in 2nd edition of the book]

A buyer wants to buy one unit of a good from an incumbent seller. The
buyer’s valuation of the good is 1, while the seller’s cost of producing it is
1/2. Before the parties trade, a rival seller enters the market and his cost, c, is
distributed on the unit interval according to a distribution function with density
g(c). The two sellers then simultaneously make price o¤ers and the buyer trades
with the seller who o¤ers the lowest price. If the two sellers o¤er the same price,
the buyer buys from the seller whose cost is lower.

1. Determine the price that the buyer pays in equilibrium, p , as a function


of c. Given p , write the payo¤s of the expected payo¤s of the buyer and
the two sellers.
2. Suppose that the distribution of c is uniform. Show p and the expected
payo¤s of the parties graphically (put c on the horizontal axis and the
equilibrium price function on the vertical axis and show the payo¤s by
pointing out the appropriate areas in the graph).

26
3. Now suppose that the incumbent seller o¤ers the buyer a contract before
the entrant shows up. The contract requires the buyer to pay the incum-
bent seller the amount m regardless of whether he buys from him or from
the entrant, and gives the buyer an option to buy from the incumbent at a
price of p (this is equivalent to giving the buyer an option to buy at a price
m + p and requiring him to pay liquidated damages of m if he switches
to the entrant). If the buyer rejects the contract things are as in part (1).
Given p and c, what is the price that the buyer will end up paying for the
good? Using your answer, write the expected payo¤s of the buyer and the
two sellers as a function of p and m.
4. Explain why the incumbent seller will choose p by maximizing the sum of
his expected payo¤, I , and the buyer’s expected payo¤s, u.
5. Write the …rst-order condition for p and show that the pro…t-maximizing
price of the incumbent seller, p , is such that p < 1=2. Also show that
if g(0) > 0 then p > 0.
6. Explain why the contract is socially ine¢ cient. Is the outcome in part (1)
socially e¢ cient? Explain the intuition for your answer.
7. Compute p assuming that the distribution of c is uniform, and show the
expected payo¤s of the parties and the social loss graphically (again, put
c on the horizontal axis and the equilibrium price function on the vertical
axis).
8. Compute p under the assumption that G(c) = c , where > 0. How
does p vary with ? Give an intuition for this result.

Solution to Exercise 24
1. In equilibrium, the price is p = 1=2 if c 1=2 (hence, the buyer buys from the
entrant), and p = c, otherwise (hence, the buyer buys from the incumbent).
Given this price, the buyer’s expected bene…t is
Z 1 Z 1
2 1
u=1 g(c)dc cg(c)dc
0 2 1
2

The expected pro…t of the incumbent seller and the entrant are:
Z 1
1
I = (c )g(c)dc;
1
2
2
Z 1
2 1
E = ( c)g(c)dc:
0 2

27
Using integration by parts, net bene…t and pro…ts are:
" Z #
1
1 1 1
u = 1 G( ) cG(c)j1=2 G(c)dc
2 2 1
2
Z 1
= G(c)dc;
1
2
1 Z 1 Z 1
1 1
I = (c )G(c) G(c)dc = G(c)dc;
2 1 1
2
2 1
2
2
1 Z 21 Z 1
1 2 2

E = (c )G(c) + G(c)dc = G(c)dc:


2 0 0 0

2. Draw …gure.
3. Given p and c, the buyer will buy from the entrant if c p and from the incum-
bent seller otherwise and will pay p. The buyer’s expected payo¤s is therefore

u=1 p m;

and the expected payo¤s of the incumbent seller and the entrant are
Z 1
1 1
I = p g(c)dc + m = p [1 G(p)] + m;
p 2 2
Z p
E = (p c)g(c)dc:
0

4. First, note that the contract must be designed so as to ensure the buyer the
same expected payo¤ as in Part 1, otherwise the buyer will reject the contract.
Second, since the incumbent seller can choose m to extract the buyer’s expected
payo¤, the expected payo¤ of the incumbent seller is equal to the sum of his
expected payo¤, I , and the buyer’s expected payo¤ u minus the expected payo¤
of the buyer absent a contract. Since the latter is a constant, p will be chosen
to maximize I + u.

5. Di¤erentiating I + u, the …rst-order condition for p is given by

1
p g(p) G(p) = 0:
2

At p = 1=2, the left-hand side of the …rst-order condition is negative. Thus,


p < 1=2. On the other hand, if g(0) > 0 then, at p = 0, the left-hand side
of the …rst-order condition is positive. Thus, p > 0. The …rst expression on
the left-hand side of the …rst-order condition is the marginal reduction in the
likelihood the buyer buys from the incumbent seller. This expression is positive
since p < 1=2: In other words, the less often the buyer exercises the option
to buy the lower are the losses to the incumbent seller from having to sell at a

28
price below his costs. The second expression on the left-hand side of the …rst-
order condition represents the marginal decreases in the sum of the expected
payo¤s of the buyer and the seller. This expression is negative because when p
increases by a dollar, the buyer pays one more dollar with probability 1, while
the incumbent seller gets an additional dollar only when the buyer buys from
him. The latter occurs with probability (1 G(p)). Thus, the total change is
G(p).
6. Since the buyer always buys the good, e¢ ciency is achieved if and only if the
lowest cost seller produces the good. In Part 1 competition ensured that this is
indeed the case. Under a contract, the incumbent produces whenever c p .
Since p < 1=2, production is ine¢ cient whenever p < c < 1=2 since then
the incumbent produces even though the entrant is more e¢ cient.

7. When c is distributed uniformly on the unit interval, the …rst-order condition


for p becomes
1
p p = 0:
2
Hence, p = 1=4. The social loss is given by
Z 1
2 1
L= c g(c)dc:
p 2

8. When G(c) = c , the …rst-order condition for p becomes

1 1
p p p = 0:
2
Solving this equation we obtain

1
p = :
2 +1
This equation shows that p is increasing with . Intuitively, an increase in
gives rise to two e¤ects: First, the higher is , the more likely is the entrant
to have high costs. Hence, the marginal cost of raising p which is G(p) (see
Part 5) becomes smaller. Second, the marginal bene…t of raising p, given by
(1=2 p)g(p), changes as well and this e¤ect may work in the opposite direction.
In our case though, the …rst positive e¤ect dominates. Hence, the less e¢ cient
the entrant is in expectation, the higher will be the price that the buyer is
required to pay.
Interestingly, this is not always true. In particularly, if G( )=g( ) is an increasing
function and there is a shift parameter that shifts both G( ) and G( )=g( )
upwards then p will decrease although the entrant is less likely to be e¢ cient.

Exercise 25 Franchising

29
A monopolistic manufacturer produces a good that is sold to three retailers.
The manufacturer has constant marginal cost equal to c < 21 . The retailers are
monopolists in three di¤erent cities. The marginal cost of the retailers is equal
to the wholesale price of the good. Each of the retailers faces a demand function
Qb = 1 b p where p is the retail and b is a variable characterizing the local
demand function. Each retailer knows the value of b in its city. Furthermore, it
is common knowledge that b = 1 in one city and b = z in two cities, z 2 (1; 2].
The manufacturer o¤ers each of the retailers a two-part tari¤ consisting of a
wholesale price w and a franchise fee f . Let us assume that the retailers accept
any contract that results in nonnegative pro…ts. The retailers choose the retail
price p in their market (i.e., there is no resale price maintenance).

1. Let w < 12 and take f as given. Find the price that a retailer sets as a
function of b. Who earns the highest pro…t, retailers with b = 1 or b = z?
2. Assume in the rest of the exercise that z = 2. Suppose …rst that the
manufacturer knows the value of b in all three cities and that z = 2. What
contract will the manufacturer o¤er to the retailers? Is the franchise fee the
same for all retailers? Is it possible for the retailer to extract all pro…ts
in the vertical chain? Assume from now on that the manufacturer does
not know the retailers’ individual b. However, the manufacturer knows
that two of the retailers have b = z = 2 and that one retailer has b = 1.
Assume also that the manufacturer wishes to serve all retailers.
3. Find the optimal franchise fee as a function of the wholesale price w.
4. Find the optimal wholesale price as a function of c. Is the wholesale price
greater or less than c?
5. Can the manufacturer extract all pro…ts by setting w and f optimally?
6. Assume now instead that c = 1=4. Show that it is optimal for the manu-
facturer only to sell to the retailer with b = 1. What happens if c ! 0?

Exercise 26 Exclusive dealing [included in 2nd edition of the book]

Suppose that two …rms produce at constant marginal costs c. There are two
periods and m buyers. Each buyer has an inverse demand curve: P (qI + qE ) =
1 (qI +qE ) where qI is the quantity sold by the incumbent and qE is the quantity
sold by the entrant. In the …rst period, there is only the incumbent in the
market. Thus, the incumbent produces the monopoly quantity. The incumbent
has marginal cost equal to cI . In the second period, an entrant with constant
marginal cost equal to zero enters into the market. The entry is foreseen by the
buyers. After entry, the two …rms compete à la Cournot. In the …rst period, the
incumbent o¤ers the buyers a fee for an exclusive dealing agreement (take-it-
or-leave-it). If a buyer accepts the o¤er, she cannot buy from the entrant in the
second period.

30
1. Suppose that the incumbent and entrant are equally e¢ cient, i.e. cI =
0. What is the maximal fee for an exclusive dealing agreement that the
incumbent is willing to o¤er to a buyer? What is the minimum fee that a
buyer is willing to accept for signing an exclusive dealing agreement? Will
there be exclusive dealing in equilibrium?
2. Consider a general marginal cost of the incumbent cI . For which values of
cI will exclusive dealing arise in equilibrium?

Solution to Exercise 26 The value an exclusive dealing is worth VI = 1=4(1


cI )2 1=9(1 2cI )2 to the incumbent. A buyer accepts if she is o¤ered a payment of,
at least, VB = 1=18(2 cI )2 1=8(1 2cI )2 to sign an exclusive dealing agreement
. Setting cI = 0 provides the solution to (1). Exclusive dealing occurs in equilibrium.
Solution to part (2): Exclusive dealing arises in equilibrium if and only if VI VB 0,
which is equivalent to cI 1=3.
Explanation: Unlike the case of Bertrand competition considered in Rasmusen et
al., exclusive dealing can arise in equilibrium under Cournot competition. The reason is
that exclusive dealing allows the incumbent and the buyer to obtain some of the rents
that the entrant would earn under no exclusive dealing. Notice: This is not possible
under Bertrand competition as the entrant earns zero pro…ts. Exclusive dealing only
arises if the incumbent is not too ine¢ cient compared to the entrant. Otherwise, the
ine¢ cient incumbent cannot o¤er a high enough fee to make it worthwhile for the
buyer to exclude the e¢ cient entrant.

Exercise 27 Long-term contracts, upgrades, and exclusion

Consider a market for a base good that is sold over two periods. In period 2,
also an upgrade may become available. For simplicity, set the mass of consumers
equal to 1 and marginal costs equal to zero. Firm 1 can be active in periods
1 and 2, …rm 2 can only be active in period 2. At the beginning of period 2,
…rms decide simultaneously whether to upgrade. Suppose that …rms maximize
the sum of pro…ts in periods 1 and 2.
The willingness-to-pay without upgrades is V per consumer in each period.
An upgrade by …rm 1 leads to a surplus of r + 1 , while an upgrade by …rm
2 would lead to a surplus of r + 2 . Firm 2 is assumed to be more e¢ cient,
2 > 1 . The upgrading cost is C. Suppose furthermore that 1 > C. This
assumption means that upgrading is socially superior to not upgrading even if
it is done by the less e¢ cient …rm.

1. Characterize the equilibrium if …rms can only o¤er short-term contracts,


i.e., …rm 1, when selling to consumers in period 1, cannot make them sign
a contract that binds consumers to buy from it in period 2.
2. Characterize the equilibrium if …rm 1 can o¤er a long-term contract that
does not allow consumers or prevents them from buying from …rm 2. Dis-
cuss your result.

31
Solutions to Exercise 27
1. Bertrand competition in period 2 implies that consumers make a net surplus of
r in period 2. In the case of short-term contracts …rm 2 upgrades in period 2
and makes a pro…t of 2 C (because, in equilibrium, …rm 1 does not upgrade).
Firm 1 makes a pro…t of r in period 1.

2. Alternatively, …rm 1 could o¤er a long-term contract requiring the consumer not
to buy from the rival in period 2. Consumers are willing to take this option if
their surplus is weakly larger than r . Hence, …rm 1 can charge r in period 1. In
addition, in period 2 it can release an upgrade and charge 1 for it. Since …rm 1’s
pro…t are r + 1 C > r , it is in the interest of …rm 1 to o¤er such a contract.
If consumers have signed with …rm 1 in period 1, …rm 2 has no incentive to
introduce an upgrade at the beginning of period 2. Long-term contracts result
because of the market power of …rm 2 in the market with upgrades. Through
long-term contracting, the more e¢ cient …rm 2 is excluded from the market.

Exercise 28 Vertical integration. [included in 2nd edition of the book]

1. Suppose that two downstream retailers sell a homogeneous product in a


downstream market. They have to pay w < 1 for each unit of the product
that they sell on to …nal consumers and do not incur any further variable
costs. The inverse downsteam market demand P (q) = 1 q , where q =
q1 + q2 . Determine the Nash equilibrium when both …rms set quantities
simultaneously.
2. Suppose that there are many such downstream markets (to be precise,
a continuum of mass 1) and that prior to the quantity setting in those
downstream markets two upstream …rms simultaneously set quantities xi .
For each unit they incur marginal costs c. Determine the subgame perfect
equilibrium of the two-stage game.
3. What would change if there is only one instead of a continuum of down-
stream markets? Is there any conceptual di¤erence between those two
settings? Explain in at most three sentences.
4. Suppose that upstream …rm 1 merges with downstream retailer 1 in each of
the many downstream markets. Suppose furthermore that …rm 1 commits
neither to sell to any downstream retailer 2 nor to buy any units from
upstream …rm 2. The timing of the game is that, at stage 1, upstream
…rms simultaneously set xi and that, at stage 2, retailers set qi . Determine
the subgame-perfect equilibrium of this two-stage game.
5. Compare your results in (2) and (4). Does the vertical merger increase the
input price for non-integrated downstream …rms? Does the vertical merger
make consumers better o¤? Explain in one or two sentences.

32
Solutions to Exercise 28
D
1. The pro…ts of each downstream …rm is i = (1 qi qj w)qi . Solving the
…rst-order conditions imposing symmetry gives q1 = q2 = (1 w)=3. Hence,
q = (1 w)2=3.
2. Since q = x, the inverse demand in the upstream market is w = 1 3x=2.
Upstream pro…ts are U i = (1 3(xi + xj )=2 c)xi . Solving for the FOCs
gives x1 = x2 = 2=9(1 c). Thus x = 4=9(1 c), w = 1=3 + (2=3)c and
p = 5=9 + (4=9)c.
3. With a continuum of downstream markets the demand by each downstream …rm
for inputs leaves the market price unchanged. With a single downstream market,
if a …rm increases its quantity this will a¤ect the input price.

4. At stage 2, downstream …rm 2 obtains the input at wholesale price w2 , whereas


downstream …rm 1 obtains the input through internal production and, thus, at
a price of 0. Thus, …rm 1’s pro…t function is D
1 = (1 q1 q2 c)q1 , while …rm
2’s pro…t function is D2 = (1 q1 q2 w2 )q2 . Solving the system of …rst-order
conditions gives q1 = (1+w2 2c)=3 and q2 = (1 2w2 +c)=3. Since q2 = x2 ,
the upstream …rm faces an inverse demand of w2 = (1 3x2 + c)=2. Its pro…t
is (w2 c)x2 . Maximizing pro…ts with respect to x2 yields x2 = 1=6(1 c)
and w2 = 1=4(1 + 3c). Hence, x1 = q1 = (5=12)(1 c). Since q = q1 + q2 =
(1=6 + 5=12)(1 c) = (7=12)(1 c), the retail price is 5=12 + (7=12)c.
5. The wholesale price paid by the non-integrated …rm is less than without the
vertical merger. This may sound surprising as upstream …rm 2 exerts monopoly
power. However, the upstream …rm faces a more elastic demand and, therefore,
charges a lower input price. In equilibrium, consumers pay a lower retail price
under vertical integration than absent vertical integration. Thus, consumers are
better o¤.

Exercise 29 Exclusive dealing and vertical integration.


Consider a vertical duopoly with exclusive dealing contracts in place, i.e.,
upstream …rm i only sells to downstream …rm { , i = 1; 2. Suppose that, at stage
1, upstream …rms and then, at stage 2, downstream …rms set prices. Downstream
demand is of the form Qi (pi ; pj ) = 1 bpi + dpj . Upstream …rms have zero
marginal costs of production and set their wholesale price. Consider subgame
perfect equilibria.
1. Characterize equilibrium upstream and downstream prices.
2. Suppose that b = d = 1. Characterize the equilibrium if …rms indexed by
1 have vertically integrated (so that the integrated …rm’s transfer price is
0).
3. Are there incentives for vertical integration (for b = d = 1)? Discuss your
results.

33
Another random document with
no related content on Scribd:
älä pois mua manaa, pyydän ma,
ja semminkin, jätä paatos!

Ole aave en aikojen menneiden,


en hautojen kouko ja peikko,
ja retoriikast' en pidä, mun on
hyvin filosofiiakin heikko.

Olen käytännön mies ma, vaitelias


ja verkkainen aina. Vaan tiedä:
mit' olet sa miettinyt mielessäs,
sen lupaan ma perille viedä.

Ja vuodet jos menköön, ma työtä teen


yhä väsymättömin voimin,
kunis todellisuutena aattees nään;
sa tuumit, ja ma, ma toimin.

Sa tuomari olet, ma pyöveli,


joka täytän tuomios määrät,
kuin tulee orjan tottelevan,
oli oikeat ne tai väärät.

Kävi Roomassa liktori kirveineen


ain' edellä konsulin ennen.
Niin sunkin, mut takana tappara käy,
nyt itses edellä mennen.

Ma olen sun liktoris, alati sua seuraan, liepeessä vaatteen


teloituskirves, mi valmis on — Teko olen ma aivojes aatteen."
VII LUKU

Menin maata, ja oli kuin tuutineet ois enkelit. Sulolta tuntuu


uni Saksan sängyissä, semminkin jos untuvapatjoille untuu.

Kuink' usein kaipasin makeutta


ma pieluksen kotimaisen
öin unettomin noilla kovilla
matrasseilla maanpakolaisen!

Hyvät untuvapatjamme maata on


ja ne hyviä unia suopi.
Niissä sielu saksalainen se maan
kaikk' ikeet yltään luopi.

Luo ikeet ja kohoo ja korkeimmat


valontaivasten tarhat tapaa.
Sielu saksalainen, kuink' untes tie
on öisten uljas ja vapaa!

Käy kalpeiksi korkeat jumalat,


kun kotkanlentoos sa rupeet!
Sun siipes kirkasti kiitäissään
jo monen tähtösen kupeet!
Maa ranskalaisten ja ryssäin on,
ja meren on herroina britit,
mut isännät ilmojen ihannemaan
on meidän Meyerit, Schmidtit.

Se on valtamaamme, siellä on
ehyt heimomme, pilvien päällä;
muut kansat kaikki on kehittyneet
maan matalan tasalla täällä. — —

Ma nukuin ja katuja noita taas,


valetuita kuutamon valoin,
olin kaikuvin askelin astuvinain
ohi vanhanaikaisten taloin.

Taas naamio silmillä takana mun


tuli tumma se seuralainen.
Mua väsytti, polvet kuin poikki ois,
mut yhä me astuimme vainen.

Yhä astuimme. Sydän mun rinnastain


oli avattu, auki rippui
se ammottavasti, virtanaan
veri sydänhaavasta tippui.

Verivirtaan monesti sormeni


ma kastoin ja pieltä oven
myös monta ohitse astuissain
ma sivusin punalla poven.

Ja aina, kun talon ma minkä noin


olin merkinnyt, kuolinsoitto
niin väräjävästi valittain
mun korvaani kumahti loitto.

Mut yöhyt synkkeni, kirkas kuu


taa peittyi kolkkojen varjoin,
rajut pilvet mustina ratsuina
ens ilmassa häilyvin harjoin.

Ja yhä olento tumma tuo


tuli perässä. Tuima terä
hihan varjosta välkki. Niin vaellettiin
me katuja hyvä erä.

Yhä mentiin, tuomiokirkkoon taas


me kunnes jo yhdyttäytiin;
ovet kaikk' oli sepposen selällään,
me kirkkoon sisälle käytiin.

Tuon holvin äärettömän vain yö


ja kuolo äänetön täytti;
joku palava lamppu se ikäänkuin
vain pimeytt' oikein näytti.

Pilaristoa kauan ma kävelin,


vain askelet saattajan vakaan
mun korviini soi yhä, jäljistäin
ei jäänyt hän askeltakaan.

Oli vihdoin paikka, min kynttilät


valas kirkkaat, mi kiviin ja kultiin
oli koristeltu; — me kappeliin
pyhän kolmen kuninkaan tultiin.
Vaan nuo pyhät kuningas-vainajat,
jotka muuten niin hiljaa nukkuu,
kas ihme! nyt arkkuinsa kannella
ne siellä kolmisin kukkuu.

Luurankoa kolme, koristein


eriskummaisin, — kurjat, valjut
pääkallot kruunua, valtikkaa
käsirangat kanteli kaljut.

Kuin nuoranuket ne heiluivat


nuo luut, jotka ammoin kuoli;
mädän löyhkää puol' oli hajussaan
ja suitsutussavua puoli.

Höpis yhden huuletkin, puheen hän


hyvin pitkän ja laajan laati;
hän selitti, miksikä minulta
hän kunnioitusta vaati.

Näet koska hän kuollut ensiksikin


ja toiseksi kuningas oli,
ja pyhimys kolmanneksi, mut ei
moneks auttanut tuo holipoli.

Vain nauravin mielin ma vastasin:


"Tuo kaikki on hukka-huolta!
Mies ollut ja mennyt sa olet, sen nään,
vaikka katsoisin miltä puolta.

Pois täältä, pois! Tilaks omansa


teille haudan on mustat mullat.
Nyt elämä ottaa haltuunsa
tään kappelin kivet ja kullat.

Ajan uuden ratsassaatto saa


ilon lyöden ja ottaa majan
tässä templissä — mielin menkää pois,
tai ma asevoimalla ajan!"

Niin lausuin ja käännähdin ja näin,


kun kamalasti se välys,
ase kamala saattajan sanattoman —
ja hän viittaukseni älys.

Kävi kirveineen hän päin ja löi


alas armahtamattomasti
nuo taikauskon katalat
luurangot pirstoiks asti.

Joka holvissa, sopessa raikui, soi


kuin tuomiopäivän pauhu!
verivirrat syöksähti rinnastain,
ja siihen mun herätti kauhu.

VIII LUKU
Kuus groshia Kölnistä Hageniin meni kuudetta riksiä
Preussin. Paha kyll' oli vaan diligenssi täys, varavaunuista
tilan ma löysin.

Syysaamu harmaa ja kostea,


loan vallassa vaunut sousi;
mut säässä ja kelissä kehnossakin
sulo tunne se suoniin nousi.

Kodin kultaisenhan se ilmaa on,


jota polttava poski tapaa!
Ja maantien rapakko onhan tuo
isänmaani rakkaan rapaa!

Hevot mulle huiskutti häntää kuin


tutut vanhat tuhansin muistoin,
kakaratkin ne kauniilta näyttivät
kuin heelmät Hesperian puistoin!

Tiellä Mühlheim on. Soma kaupunki


ja uurasta, hiljaista väki.
Oli toukokuu kolmekymmentä-yks,
kun viimeks sen silmäni näki.

Ilos silloin ilmassa päivän koi,


maa kukkaishäitä vietti,
kevätkaihoja linnut liversi,
ja ihmiset toivoi ja mietti —

ne mietti: "Pian jo pääsemme noista laihoista ritareista,


eromaljan pitkistä mittaamme heille teräspikareista!
Saa Vapaus liehuvin lippuineen,
käy karkelot, soitot soipi,
ehkä Bonapartenkin takaisin
se tuonelta tuoda voipi!"

Hyvä Jumala! tääll' ovat ritarit vaan,


ja moni nulikka-paha,
joka itikkahoikkana maahan sai,
nyt on mahtaja isomaha.

Nuo kalpeat kanaljat, naamaltaan


kuin rakkaus, toivo ja usko,
heillä meidän viljoista viineist' on
nyt nokalla rohkea rusko. — —

Eikä juosta ja rynnätä Vapaus


voi enää, sen jalka on nilkku;
alas Pariisin torneista tuijottaa
vain trikolori-tilkku.

Ylös sittemmin keisari noussut on,


mut mieheksi äänettömäksi
madot Englannin hänet jo saattoi nuo,
hän jälleen hautaan läksi.

Omin silmin ne hautajaiset näin,


näin kultaiset vaunut, joilla
kuus voitotart' oli kultaista,
arkku kultainen hartioilla.

Kautt' Elyseisten kenttien,


kautt' uljaan Riemukaaren,
kautt' usmain ja lunten hiljaa niin
kävi saatto sankaripaaren.

Rämis vihloin kauhea musiikki,


väris soittajat viiman alla.
Sotalippujen kotkat tervehti
mua katseella murehtivalla.

Niin oudoilta, vanhaan muisteloon


lumotuilta ihmiset näytti —
tarun keisarillisen kangastus
taas mielet tenhoi ja täytti.

Sinä päivänä itkin. Kyyneltä en silmästä estää voinut, kun


kajahti huuto "Vive l'Empereur!" tuo armas, ammoin soinut.
IX LUKU

Ma neljännest' ennen kahdeksaa olin aamulla Kölnistä tiellä;


tulin kolmen jo korvilla Hageniin, on päivällispaikka siellä.

Oli pöytä katettu. Herkut näin


ma taas peri-germanilaiset.
Sua hapankaalini tervehdin,
sun lemus on ihanaiset!

Kera kaalisten haudotut kastanjat!


niit' ennen söin emon koissa! —
Kapaturskat te kotoiset, terve myös!
miten viisaina virutte voissa!

Isänmaa joka ihmissydämen


on ainainen aarre — maistaa
myös hyvältä munat ja silakat,
kun hyvin ne paahtaa ja paistaa.

Miten riemusi makkarat rasvassaan!


Ja rastaat, paistetut, pienot
herran-enkelit omenahilloineen,
tervetullut! ne visersi vienot.
"Tervetullut, maanmies!" — ne visersi —
"jo viikon viivyitkin poissa,
kera vierasten lintujen leikamoit
vain vieraissa viidakoissa!"

Oli pöydässä hanhi, hiljainen,


hyvänsävyinen olento siellä.
Mua kerran lempinyt lie kenties,
kun nuor' oli kumpikin vielä.

Mua silmäs se katsein niin merkitsevin,


niin syvin, niin hellin, niin heikein!
Oli kaunis sisällä sielu kai,
mut liha ei murennut leikein.

Tinavadissa pöytään vaeltavan näin sianpään myös rehdin;


siankärsät ne meillä koristetaan yhä vielä laakerinlehdin.
X LUKU

Heti Hagenin takana tapasi yö, ja puistatus päälleni pakkas


omituinen. Se vasta Unnassa majatalomme lämmössä
lakkas.

Siellä mulle kiltisti punssia kaas


soma tyttö tyllykkä muudan —
kuin keltasilkkiä kiharat,
ja silmät kuin lempeä kuudan.

Tuo westfalilainen soperrus


taas soittona korvissa soi mun.
Sulomuistoja punssi se höyrysi,
tutut veikot mielehen toi mun,

kera joitten ma Göttingenissä join jo kerran sen


kymmenenkin, kunis toistemme povelle langettiin ja alle
pöydän ja penkin!

Niin paljon heist' ain' olen pitänyt


kelpo Westfalin pojista noista —
väki niin luja, vakaa ja uskollinen,
ei koskaan kuorella loista.
Miten miekkasill' uljaina seisoivat
he leijonamielin ja -tarmoin!
Niin vilpittömästi he iskivät,
niin kvartein ja terssein varmoin.

Hyvin taistelevat, hyvin maistelevat,


ja silmiss' on kyynellammet,
kun ystäväliittoon he kättä lyö,
nuo sentimentaaliset tammet.

Sua kaitkoon taivas ja kartuttakoon,


jalo kansa, sun kylvöjes keot,
sodat maaltas ja maineet estäköön
ja uroot ja urosten teot.

Läpi tutkinnon suokoon solahtaa joka poikas, ja morsiamen


joka tyttärestäsi laittakoon lain jälkeen aina — amen!
XI LUKU

Siis Teutoburgin on metsä tää, josta Tacitus tarinoipi, tää siis


on se klassillinen suo, johon tarttui Varuksen koipi.

Jalo Hermann, kheruskein ruhtinas


tässä häviöön hänet hääsi;
tässä ravassa Saksan kansallisuus
se voiton päälle pääsi.

Jos Hermann vaalevin heimoineen


ois tässä tappion saanut,
me roomalaisia oltais nyt,
ois Saksan vapaus laannut!

Tavat Rooman täällä nyt vallitseis,


ja Rooman kieli se soisi;
vestaaleja Münchenissäkin ois,
qviriittejä svaabit oisi!

Nyt Hengstenberg olis haruspex


ja sonnein suolia urkkis.
Neander, neuvokas auguuri,
se lintuin kulkua kurkkis.
Birch-Pfeiffer tärpättiä jois
kuin muinen naiset Rooman
(uriinille kuuluu se antaneen
niin erinomaisen arooman).

Ei lumppusaksa nyt Raumer ois,


vaan roomalainen Lumpacius,
ja riimittä laulais Freiligrath
kuin muinen Flaccus Horatius.

Isä Jahn, tuo ruokoton kerjuri,


ois nimeltä Grobianus.
Puhuis Massmann, me hercule! latinaa,
tuo Marcus Tullius Massmanus!

Jalopeurain, hyenain, shakaalein


kera otella totuuden tulkit
areenalla sais, katulehdiss' ei
tulis kintuille penikka-nulkit.

Yksi Nero, ei kolmea tusinaa


maanisää meill' olis tuhmaa.
Me valtimot avaten näytettäis
orjuuden vahdeille uhmaa.

Ois Schelling maar koko Seneca,


kun kohtais moinen fatum.
Corneliuksemme kuulla sais:
"Non pictum est, qvod cacatum!" —

Vaan onneksi Varuksen legiot


löi Hermann taistelollaan,
pois roomalaiset ne tungettiin,
ja me saksalaisia ollaan!

Meillä Saksan mieli ja kieli on,


kuin ollut on iät päivät;
nimi aasin on aasi, ei asinus,
ja svaabit svaabeiksi jäivät.

On Raumer lumppusaksa vain,


saa kotkantähden viimein.
Ei ole Horatius Freiligrath,
vain laatii lauluja riimein.

Puhu onneks ei Massmann latinaa,


Birch-Pfeiffer vain draamoja valaa,
tärpättiä saastaista särpi ei
kuin Rooman naikkoset salaa.

Oi Hermann, se on sun ansiosi


Siks sulle patsas suuri
Detmoldiin tehdään — se kohtuus on
panin lisäni listaan juuri.
XII LUKU

Läpi öisen metsän vaunumme meni, nytki — yht'äkkiä rysäys,


ja pyörä irti. Ei erittäin juur' ollut hauska se pysäys.

Alas astui ajaja laudaltaan


avun hakuun kylästä kiirein.
Jäin yksin, metsä mun ympäröi
sydänöisin, ulvovin piirein.

Sudet nälistyneillä äänillään


ne ulvoi viiltävin vimmoin.
Kuin kynttilät yössä tuikkivat
nuo silmät kiiluvin kimmoin.

Kai oli kuullut mun tulostain


tuo metsän karvainen kansa,
ja kunniakseni tulitti tien
ja kaiutti kuorojansa.

Mulle kunniavastaanotto, sen nään,


noin suodaan silmillä, suilla!
Otin asennon heti ja vastaamaan
kävin liikkeillä liikutetuilla:
"Sudet, veljet! Onnekas hetki tää,
joka jaloon seuraanne toi mun,
niin sydämenpohjainen ulvonta
missä tervehdykseksi soi mun.

Oi, mittaamaton on tunne tuo,


minkä mielelleni se antaa;
tään hetken ihanan iäti
ma tahdon muistossa kantaa.

Teitä luottamuksesta kiittää saan,


jota minulle olette suoneet
ja koetuksissa näkyviin
ain' uskollisesti tuoneet.

Sudet, veljet! Te minuun luotitte,


ette käyneet kurjien paulaan,
muka ett' olin koiraksi kääntynyt
ja ottanut renkaan kaulaan,

pian luopio muka ett' ilmestyis hovineuvosna


lammaslaumaan — alapuoll' oli arvoni astuakin mun moisista
puolustaumaan.

Jos lampaanturkin mä toisinaan


vedin päälleni pahaan säähän,
mun haaveilla onnea lammasten
toki tullut ei ikinä päähän.

En ole lammas, en koira, en


hovineuvos, en silli mikään —
susi oon, suden mulla on sydän ja suu
ja ikenet niin-ikään.

Olen susi ja sutten keralla


ain' ulvon ma — teit' en heitä.
Niin, itseänne kun autatte,
niin luojakin auttaa teitä!"

Se oli mun puheeni, sellaisnaan kuin hetki mieleeni toi sen!


"Yleisiin Sanomiinsa" Kolb typisteltynä kopioi sen.
XIII LUKU

Nous aurinko Paderbornissa kovin nuivalla katsannolla. Sen


viraton virka on tosiaan, maan tyhmän lamppuna olla!

Kun yhden kupeen valaistuaan


se hehkuvin kiirein heittää
valon toiselle puolle, niin pimeys
sill' aikaa jo toisen peittää.

Kivi Sisyphon yhä se vierii pois,


Danaiidit seulaansa täyttää
ei saata, ja turhaan aurinko
maapallolle soihtua näyttää! — —

Tien varrella, valoon kun alkavaan


sumu hälveni aamun suussa,
sen miehen kuvan ma huomasin,
joka riippui ristinpuussa.

Näkös, serkku parka, mun murheiseks


saa aina, jok' alttihisti
tämän maailman mielit lunastaa,
sa narri, sa utopisti!

You might also like